2020 AMC 10A Problems/Problem 21

Revision as of 23:10, 31 January 2020 by Seanyoon777 (talk | contribs) (Solution)

There exists a unique strictly increasing sequence of nonnegative integers $a_1 < a_2 < … < a_k$ such that\[\frac{2^{289}+1}{2^{17}+1} = 2^{a_1} + 2^{a_2} + … + 2^{a_k}.\]What is $k?$

$\textbf{(A) } 117 \qquad \textbf{(B) } 136 \qquad \textbf{(C) } 137 \qquad \textbf{(D) } 273 \qquad \textbf{(E) } 306$

Solution

First, replace $2^(17)$ as $a$. Then, the given equation becomes $(a^(17)+1)/(a+1)=a^(16)-a^(15)+a^(14)...-a^1+a^0$. Now consider only $a^(16)-a^15$. This equals $a^(15)(a-1)=a^15*(2^(17)-1)$. Note that $2^(17)-1$ equals $2^(16)+2^15+...+1$, since the sum of a geometric sequence is $(a^n-1)/(a-1)$. Thus, we can see that $a^(16)-a^(15)$ forms the sum of 17 different powers of 2. Applying the same thing to each of $a^(14)-a^13$, $a^12-a^11$, and so on. This gives us $17*8=136$. Our answer is $\boxed{\textbf{(B) } 136}$.

~seanyoon777

See Also

2020 AMC 10A (ProblemsAnswer KeyResources)
Preceded by
Problem 20
Followed by
Problem 22
1 2 3 4 5 6 7 8 9 10 11 12 13 14 15 16 17 18 19 20 21 22 23 24 25
All AMC 10 Problems and Solutions

The problems on this page are copyrighted by the Mathematical Association of America's American Mathematics Competitions. AMC logo.png